Đến nội dung

Hình ảnh

Bổ đề hoán vị

* * * - - 2 Bình chọn

  • Please log in to reply
Chủ đề này có 7 trả lời

#1
Nguyenhuyen_AG

Nguyenhuyen_AG

    Trung úy

  • Thành viên nổi bật 2016
  • 945 Bài viết

*
Phổ biến

Chào các thành viên của VMF, nhân dịp Tết Bính Thân 2016 mình xin tặng cho các thành viên trên VMF một chuyên đề bất đẳng thức của mình. Chuyên đề này ... à mà thôi mình dở văn nên xin bỏ qua đoạn này, mọi người đọc và cho tác giả ý kiến. :)

 

BỔ ĐỀ HOÁN VỊ

 

1. Mở đầu

 

Năm 2008 trên diễn đàn toán học Art of Problem Solving anh Võ Quốc Bá Cẩn đề xuất một bổ đề khá thú vị sau:

Bổ đề. Với mọi số thực dương $a,\,b,\,c$ thỏa mãn $a+b+c=1,$ đặt $q = ab+bc+ca\,(1 \geqslant 3q).$ Chứng minh rằng
\begin{equation} \label{(bodehoanvi)}
\frac{a}{b}+\frac{b}{c}+\frac{c}{a} \geqslant \frac{2(27q^2-9q+1)}{9q^2-2q+(1-3q)\sqrt{q(1-3q)}}+\frac{1}{q}-6.
\end{equation}

 

(Võ Quốc Bá Cẩn)

 

Lời giải. Đặt $p=a+b+c,\,q=ab+bc+ca$ và $r=abc$ (ta sẽ thống nhất cách đặt này cho cả bài viết) khi đó
\[(a-b)^2(b-c)^2(c-a)^2 = p^2q^2 - 4q^3 + 2p(9q - 2p^2)r - 27r^2.\]
Ta có đánh giá
\[\begin{aligned} 2(ab^2+bc^2+ca^2) & = \sum ab(a+b) + (a-b)(b-c)(c-a) \\ &= pq - 3r + (a-b)(b-c)(c-a) \\& \geqslant pq - 3r - \sqrt{(a-b)^2(b-c)^2(c-a)^2} \\& = pq - 3r - \sqrt{p^2q^2 - 4q^3 + 2p(9q - 2p^2)r - 27r^2},\end{aligned}\]
vì thế
\[ab^2+bc^2+ca^2 \geqslant \frac{pq - 3r - \sqrt{p^2q^2 - 4q^3 + 2p(9q - 2p^2)r - 27r^2}}{2}.\]
Mặt khác từ giả thiết ta được $p = 1$ cho nên
\[\frac{a}{b}+\frac{b}{c}+\frac{c}{a} = \frac{ab^2+bc^2+ca^2}{abc} \geqslant \frac{q-3r-\sqrt{q^2-4q^3+2(9q-2)r-27r^2}}{2r}.\]
Xét hàm số
\[f\,(r) = \frac{q-3r-\sqrt{q^2-4q^3+2(9q-2)r-27r^2}}{2r},\]
tính đạo hàm
\[f^{'}(r) = \frac{q^2-4q^3+(9q-2)r-q\sqrt{q^2-4q^3+2(9q-2)r-27r^2}}{2r^2\sqrt{q^2-4q^3+2(9q-2)r-27r^2}},\]
do đó phương trình $f^{'}(r) = 0$ có nghiệm
\[r = r_0 = \frac{q^2\left[9q^2-2q+(1-3q)\sqrt{q(1-3q)}\right]}{27q^2-9q+1}.\]
Lập bảng biến thiên ta được $f(r) \geqslant f(r_0).$ Biến đổi
\[\begin{aligned} f(r_0) & = \frac{q - 3r_0 - \sqrt{q - 4q^3 + 2(9q - 2)r_0 - 27r^2_0}}{2r_0} \\& = \frac{q-3r_0 - \dfrac{q^2 - 4q^3 + (9q - 2)r_0}{q}}{r_0} \\& = \frac{2q^3 + (1 - 6q)r_0}{qr_0} = \frac{2q^2}{r_0} + \frac{1}{q} - 6 \\& = \frac{2(27q^2 - 9q + 1)}{9q^2 - 2q + (1 - 3q)\sqrt{q(1 - 3q)}} + \frac{1}{q} - 6.\end{aligned}\]
Đây chính là điều phải chứng minh.

Nhận xét.

 

  1. Ở thời điểm đó bổ đề trên là một bài toán rất khó, do hình thức khá cồng kềnh, không đẹp mắt (có căn thức nằm ở mẫu số) nên không nhận được sự quan tâm của nhiều người, sau này vào năm 2011 anh Lê Hữu Điền Khuê (Nesbit) mới đưa ra một chứng minh khác trên Diễn Đàn Toán Học.
  2. Đây là một kết quả rất chặt với vô số các trường hợp để đẳng thức xảy ra và cũng là dạng chặt nhất trong lớp các bài toán có dạng \[f\left(\frac{a}{b},\,\frac{b}{c},\,\frac{c}{a},\,a+b+c,\,ab+bc+ca\right) \geqslant 0.\]
  3. Bổ đề này sẽ giúp chúng ta sẽ giải quyết được rất nhiều bài toán khó sau đây (đã từng là unsolve suốt một thời gian dài trên AoPS).

2. Các bài toán áp dụng

Bài toán 1. Cho $a,\,b,\,c$ là ba số thực dương. Chứng minh rằng
\[\frac{a}{b}+\frac{b}{c}+\frac{c}{a} + \frac{28(ab+bc+ca)}{(a+b+c)^2} \geqslant 12.\]

 

(Võ Quốc Bá Cẩn)

 

Lời giải. Do tính thuần nhất của bài toán nên ta có thể chuẩn hóa $p=1,$ khi đó
\[\frac{28(ab+bc+ca)}{(a+b+c)^2} = \frac{28q}{p^2} = 28q.\]
Áp dụng bổ đề \eqref{(bodehoanvi)} ta đưa bài toán về chứng minh
\[\frac{2(27q^2-9q+1)}{9q^2-2q+(1-3q)\sqrt{q(1-3q)}} + \frac{1}{q} - 6 + 28q \geqslant 12,\]
hay là
\begin{equation} \label{lab1}
\frac{2(27q^2-9q+1)}{9q^2-2q+(1-3q)\sqrt{q(1-3q)}} + \frac{1}{q} + 28q \geqslant 18.
\end{equation}
Bất đẳng thức lúc này chỉ còn một biến nên hai công cụ đầu tiên mà chúng ta nghĩ đến là khảo sát hàm hoặc quy đồng phân tích nhân tử. Tuy nhiên biểu thức này khi lấy đạo hàm sẽ cho ra một kết quả “rất khủng” còn nếu phân tích nhân tử thì thì biểu thức thu được cũng không khá hơn mấy vì sự xuất hiện của căn thức ở mẫu.

Viết \eqref{lab1} lại như sau
\begin{equation} \label{lab2}
\frac{2(27q^2 - 9q + 1)}{9q^2 - 2q + (1 - 3q) \cdot q \cdot \sqrt{\frac{1 - 3q}{q}}} + \frac{1}{q} + 28q \geqslant 18.
\end{equation}
Đặt $x = \sqrt{\frac{1-3q}{q}} \geqslant 0$ thì $q = \frac{1}{x^2+3},$ bất đẳng thức \eqref{lab2} trở thành
\[\frac{2(x^2+3x+3)}{x+1} + x^2 + \frac{28}{x^2+3} \geqslant 15.\]
Xét hiệu hai vế ta được
\[\frac{2(x^2+3x+3)}{x+1} + x^2 + \frac{28}{x^2+3} - 15 = \frac{(x^3+5x^2+3x+1)(x-1)^2}{(x+1)(x^2+3)} \geqslant 0.\]
Đẳng thức xảy ra khi và chỉ khi và chỉ khi
\[\frac{a}{\sqrt{7}-\tan\frac{\pi}{7}} = \frac{b}{\sqrt{7}-\tan\frac{2\pi}{7}} = \frac{c}{\sqrt{7}-\tan\frac{4\pi}{7}}.\]
Vậy ta có điều phải chứng minh.

Bài toán 2. Cho $a,\,b,\,c$ là ba số thực dương. Chứng minh rằng
\[\frac{a}{b}+\frac{b}{c}+\frac{c}{a} + \frac{7(ab+bc+ca)}{a^2+b^2+c^2} \geqslant \frac{17}{2}.\]

 

(Võ Quốc Bá Cẩn)

 

Lời giải. Tương tự như trên ta cũng chuẩn hóa $p=1,$ khi đó
\[\frac{7(ab+bc+ca)}{a^2+b^2+c^2} = \frac{7q}{p^2-2q} = \frac{7q}{1-2q}.\]
Áp dụng bổ đề \eqref{(bodehoanvi)} ta đưa bài toán về chứng minh
\[\frac{2(27q^2-9q+1)}{9q^2-2q+(1-3q)\sqrt{q(1-3q)}} + \frac{1}{q} - 6 + \frac{7q}{1-2q} \geqslant \frac{17}{2},\]
hay là
\begin{equation} \label{lab3}
\frac{2(27q^2 - 9q + 1)}{9q^2 - 2q + (1 - 3q) \cdot q \cdot \sqrt{\frac{1 - 3q}{q}}} + \frac{1}{q} + \frac{7q}{1-2q} \geqslant \frac{29}{2}.
\end{equation}
Đặt $x = \sqrt{\frac{1-3q}{q}} \geqslant 0$ thì $q = \frac{1}{x^2+3},$ bất đẳng thức \eqref{lab3} trở thành
\[\frac{2(x^2+3x+3)}{x+1}+x^2+\frac{7}{x^2+1} \geqslant \frac{23}{2},\]
tương đương với
\[\frac{(2x^3+10x^2+9x+3)(x-1)^2}{2(x+1)(x^2+1)} \geqslant 0.\]
Đẳng thức xảy ra khi và chỉ khi
\[\frac{a}{\sqrt{7}-\tan\frac{\pi}{7}} = \frac{b}{\sqrt{7}-\tan\frac{2\pi}{7}} = \frac{c}{\sqrt{7}-\tan\frac{4\pi}{7}}.\]
Bài toán được chứng minh.

 

Nhận xét. Bài toán này là hệ quả của bài toán 1. Thật vậy vì
\[\frac{a}{b}+\frac{b}{c}+\frac{c}{a} \geqslant 12 - \frac{28(ab+bc+ca)}{(a+b+c)^2},\]
nên ta chỉ cần chứng minh
\[12 - \frac{28(ab+bc+ca)}{(a+b+c)^2} + \frac{7(ab+bc+ca)}{a^2+b^2+c^2} \geqslant \frac{17}{2},\]
hay là
\[\frac{7(a+b+c)^2}{2(a^2+b^2+c^2)} \geqslant \frac{28(ab+bc+ca)}{(a+b+c)^2},\]
hoặc
\[8(ab+bc+ca)(a^2+b^2+c^2) \leqslant (a+b+c)^4.\]
Áp dụng bất đẳng thức AM-GM, ta có
\[\begin{aligned}8(ab+bc+ca)(a^2+b^2+c^2) \leqslant  \left[2(ab+bc+ca) + (a^2+b^2+c^2)\right]^2  = (a+b+c)^4.\end{aligned}\]
Khi đẳng thức xảy ra thì ta được đẳng thức khá đẹp mắt $a^2+b^2+c^2 = 2(ab+bc+ca).$

Bài toán 3. Cho $a,\,b,\,c$ là ba số thực dương. Chứng minh rằng
\[\left(\frac{a}{b}+\frac{b}{c}+\frac{c}{a}\right)^2 + \frac{70(ab+bc+ca)}{a^2+b^2+c^2} \geqslant 60.\]

 

(Nguyễn Văn Huyện)

 

Lời giải. Chuẩn hóa $p=1$ và áp dụng bổ đề \eqref{(bodehoanvi)} ta đưa bài toán về chứng minh
\begin{equation} \label{lab3.1}
\left[\frac{2(27q^2-9q+1)}{9q^2-2q+(1-3q)\sqrt{q(1-3q)}} + \frac{1}{q} - 6\right]^2 + \frac{70q}{1-2q} \geqslant 60.
\end{equation}
Đặt $x = \sqrt{\frac{1-3q}{q}} \geqslant 0$ thì $q = \frac{1}{x^2+3},$ bất đẳng thức \eqref{lab3.1} trở thành
\[\left(\frac{x^3+3x^2+3x+3}{x+1}\right)^2 + \frac{70}{x^2+1} \geqslant 60,\]
tương đương với
\[\frac{(x^6+8x^5+31x^4+84x^3+119x^2+76x+19)(x-1)^2}{(x^2+1)(x+1)^2} \geqslant 0.\]
Đẳng thức xảy ra khi và chỉ khi
\[\frac{a}{\sqrt{7}-\tan\frac{\pi}{7}} = \frac{b}{\sqrt{7}-\tan\frac{2\pi}{7}} = \frac{c}{\sqrt{7}-\tan\frac{4\pi}{7}}.\]
Bài toán được chứng minh.

Bài toán 4. Cho $a,\,b,\,c$ là ba số thực dương. Chứng minh rằng
\[\left(\frac{a}{b}+\frac{b}{c}+\frac{c}{a}\right)^2 + \frac{280(ab+bc+ca)}{(a+b+c)^2} \geqslant 95.\]

 

(Tạ Hồng Quảng)

 

Lời giải. Chuẩn hóa $p=1$ và áp dụng bổ đề \eqref{(bodehoanvi)} ta đưa bài toán về chứng minh
\begin{equation} \label{lab3.2}
\left[\frac{2(27q^2-9q+1)}{9q^2-2q+(1-3q)\sqrt{q(1-3q)}} + \frac{1}{q} - 6\right]^2 + 280q \geqslant 95.
\end{equation}
Đặt $x = \sqrt{\frac{1-3q}{q}} \geqslant 0$ thì $q = \frac{1}{x^2+3},$ bất đẳng thức \eqref{lab3.2} trở thành
\[\left(\frac{x^3+3x^2+3x+3}{x+1}\right)^2 + \frac{280}{x^2+3} \geqslant 95,\]
tương đương với
\[\frac{(x^6+8x^5+33x^4+100x^3+144x^2+88x+22)(x-1)^2}{(x^2+3)(x+1)^2} \geqslant 0.\]
Đẳng thức xảy ra khi và chỉ khi
\[\frac{a}{\sqrt{7}-\tan\frac{\pi}{7}} = \frac{b}{\sqrt{7}-\tan\frac{2\pi}{7}} = \frac{c}{\sqrt{7}-\tan\frac{4\pi}{7}}.\]
Bài toán được chứng minh.

Bài toán 5. Cho $a,\,b,\,c$ là ba số thực dương. Chứng minh rằng
\[\frac{a}{b}+\frac{b}{c}+\frac{c}{a} \geqslant \frac{a^2+b^2+c^2}{ab+bc+ca} + \frac{6(a^2+b^2+c^2)}{(a+b+c)^2}.\]

 

(Nguyễn Văn Quý)

 

Lời giải. Chuẩn hóa $p=1$ và áp dụng bổ đề \eqref{(bodehoanvi)} ta đưa bài toán về chứng minh
\begin{equation} \label{lab8}
\frac{2(27q^2-9q+1)}{9q^2-2q+(1-3q)\sqrt{q(1-3q)}} + \frac{1}{q} - 6 \geqslant \frac{1-2q}{q} + 6(1-2q).
\end{equation}
Đặt $x = \sqrt{\frac{1-3q}{q}} \geqslant 0$ thì $q = \frac{1}{x^2+3},$ bất đẳng thức \eqref{lab8} trở thành
\[\frac{x^3+3x^2+3x+3}{x+1} \geqslant \frac{(x^2+9)(x^2+1)}{x^2+3},\]
hay là
\[\frac{2x^2(x-1)^2}{(x+1)(x^2+3)} \geqslant 0.\]
Đẳng thức xảy ra khi và chỉ khi $a=b=c$ hoặc
\[\frac{a}{\sqrt{7}-\tan\frac{\pi}{7}} = \frac{b}{\sqrt{7}-\tan\frac{2\pi}{7}} = \frac{c}{\sqrt{7}-\tan\frac{4\pi}{7}}.\]
Chứng minh hoàn tất.

Bài toán 6. Cho $a,\,b,\,c$ là ba số thực dương. Chứng minh rằng
\[\frac{a}{b}+\frac{b}{c}+\frac{c}{a}+\frac{k(ab+bc+ca)}{a^2+b^2+c^2} \geqslant 3+k,\]
trong đó $k = 3\sqrt[3]{4} - 2.$

 

(Ji Chen)

Lời giải. Chuẩn hóa $p=1$ và áp dụng bổ đề \eqref{(bodehoanvi)} ta đưa bài toán về chứng minh
\begin{equation} \label{lab4}
\frac{2(27q^2-9q+1)}{9q^2-2q+(1-3q)\sqrt{q(1-3q)}} + \frac{1}{q} + \frac{kq}{1-2q} \geqslant 9 + k.
\end{equation}
Đặt $x = \sqrt{\frac{1-3q}{q}} \geqslant 0$ thì $q = \frac{1}{x^2+3}$ bất đẳng thức \eqref{lab4} trở thành
\[\frac{2(x^2+3x+3)}{x+1}+x^2+\frac{k}{x^2+1} \geqslant k+6,\]
hay là
\[\frac{x^2[x^3+3x^2+(1-k)x-k+3]}{(x+1)(x^2+1)} \geqslant 0.\]
Bất đẳng thức này đúng vì với $k = 3\sqrt[3]{4} - 2,$ thì
\[x^3+3x^2+(1-k)x-k+3 = \big(x + 1 + 2\sqrt[3]{2}\big)\big(x + 1 - \sqrt[3]{2} \big)^2 \geqslant 0.\]
Đẳng thức xảy ra khi và chỉ khi $x=0$ hoặc $x = \sqrt[3]{2} - 1,$ cụ thể

  • Nếu $x = 0$ thì $q = \frac{1}{3}$ kết hợp với $p = 1$ ta được $a = b = c.$
  • Nếu  $x = \sqrt[3]{2} - 1$ thì $q = \frac{2+\sqrt[3]{2}}{10}$ dẫn đến $r = \frac{3-\sqrt[3]{2}}{50},$ kết hợp với $p=1$ ta thấy $a,\,b,\,c$ lần lượt là ba nghiệm của phương trình \[t^3 - t^2 + \frac{2+\sqrt[3]{2}}{10}t - \frac{3-\sqrt[3]{2}}{50} = 0.\] Bằng Maple hoặc Wolframalpha ta tìm được \[\left\{ \begin{aligned} & a = \frac{1}{3}+\frac{1}{3}\sqrt{\frac{8-6\sqrt[3]{2}}{5}} \cos \left(\frac{1}{3} \arccos \sqrt{\frac{101-54\sqrt[3]{4}}{20}}\right) \\ & b =  \frac{1}{3}-\frac{1}{3}\sqrt{\frac{8-6\sqrt[3]{2}}{5}} \sin \left(\frac{\pi}{6}-\frac{1}{3} \arccos \sqrt{\frac{101-54\sqrt[3]{4}}{20}}\right)\\ & c = \frac{1}{3}-\frac{1}{3}\sqrt{\frac{8-6\sqrt[3]{2}}{5}} \sin \left(\frac{\pi}{6}+\frac{1}{3} \arccos \sqrt{\frac{101-54\sqrt[3]{4}}{20}}\right)\end{aligned}\right.\] cùng các hoán vị.

Bài toán được chứng minh.

Bài toán 7. Cho $a,\,b,\,c$ là ba số thực dương. Chứng minh rằng
\[\frac{a}{b}+\frac{b}{c}+\frac{c}{a}+k\geqslant \frac{(9+3k)(a^2+b^2+c^2)}{(a+b+c)^2},\]
với $k = 3\sqrt[3]{2} - 3.$

(Võ Quốc Bá Cẩn, Bách Ngọc Thành Công)

Lời giải. Chuẩn hóa $p=1$ và áp dụng bổ đề \eqref{(bodehoanvi)} ta đưa bài toán về chứng minh
\begin{equation} \label{lab5}
\frac{2(27q^2-9q+1)}{9q^2-2q+(1-3q)\sqrt{q(1-3q)}} + \frac{1}{q} - 6 + k \geqslant (9+3k)(1-2q).
\end{equation}
Đặt $x = \sqrt{\frac{1-3q}{q}} \geqslant 0$ thì $q = \frac{1}{x^2+3},$ bất đẳng thức \eqref{lab5} trở thành
\[\frac{x^3+3x^2+3x+3}{x+1} + k \geqslant \frac{(9+3k)(x^2+1)}{x^2+3}.\]
Với $k = 3\sqrt[3]{2} - 3,$ ta có
\[\frac{x^3+3x^2+3x+3}{x+1} + k-\frac{(9+3k)(x^2+1)}{x^2+3} = \frac{x^2\big(x+1+2\sqrt[3]{4}\big)\big(x+1-\sqrt[3]{4}\big)^2}{(x+1)(x^2+3)} \geqslant 0\]
Đẳng thức xảy ra khi và chỉ khi $a=b=c$ hoặc $a,\,b,\,c$ lần lượt là ba nghiệm của phương trình
\[t^3 - t^2 + \frac{2+\sqrt[3]{4}}{12} t - \frac{1}{36} = 0,\]
cụ thể
\begin{equation} \label{nghiembac3}
\left\{ \begin{aligned} & a = \frac{1}{3}+\frac{1}{3}\sqrt{2-\sqrt[3]{4}} \cos \left(\frac{1}{3} \arccos \sqrt{\frac{7-\sqrt[3]{2}}{4}}\right) \\ & b = \frac{1}{3} - \frac{1}{3}\sqrt{2-\sqrt[3]{4}} \sin \left(\frac{\pi}{6}-\frac{1}{3} \arccos \sqrt{\frac{7-\sqrt[3]{2}}{4}}\right) \\ & c = \frac{1}{3} - \frac{1}{3}\sqrt{2-\sqrt[3]{4}} \sin \left(\frac{\pi}{6}+\frac{1}{3} \arccos \sqrt{\frac{7-\sqrt[3]{2}}{4}}\right)\end{aligned}\right.
\end{equation}
cùng các hoán vị. Bài toán được chứng minh.

Bài toán 8. Cho $a,\,b,\,c$ là các số thực dương. Chứng minh rằng
\[\frac{a}{b}+\frac{b}{c}+\frac{c}{a} \geqslant \sqrt{9-k+\frac{k(a^2+b^2+c^2)}{ab+bc+ca}},\]
trong đó $k = 3\big(1+\sqrt[3]2\big)^2.$

(Võ Quốc Bá Cẩn)

Lời giải. Chuẩn hóa $p=1$ và áp dụng bổ đề \eqref{(bodehoanvi)} ta đưa bài toán về chứng minh
\begin{equation} \label{lab6}
\frac{2(27q^2-9q+1)}{9q^2-2q+(1-3q)\sqrt{q(1-3q)}} + \frac{1}{q} - 6 \geqslant \sqrt{9-k+\frac{k(1-2q)}{q}}.
\end{equation}
Đặt $x = \sqrt{\frac{1-3q}{q}} \geqslant 0$ thì $q = \frac{1}{x^2+3},$ bất đẳng thức \eqref{lab6} trở thành
\[\frac{2(x^2+3x+3)}{x+1}+x^2-3 \geqslant \sqrt{9 + kx^2},\]
hay là
\[\frac{x^3+3x^2+3x+3}{x+1} \geqslant \sqrt{9 + kx^2},\]
hoặc
\[(x^3+3x^2+3x+3)^2 \geqslant (9 + kx^2)(x+1)^2,\]
\[x^2[x^4+6x^3+(15-k)x^2+2(12-k)x-k+18] \geqslant 0.\]
Đặt
\[P = x^4+6x^3+(15-k)x^2+2(12-k)x-k+18,\]
ta sẽ chứng minh $P \geqslant 0.$ Thật vậy, với $k = 3\big(1+\sqrt[3]2\big)^2$ thì
\[P = \big(x+1-\sqrt[3]{2}+\sqrt[3]{4}\big)\big(x+3+\sqrt[3]{2}+\sqrt[3]{4}\big)\big(x+1-\sqrt[3]{4}\big)^2 \geqslant 0.\]
Đẳng thức xảy ra khi và chỉ khi $a=b=c$ hoặc $a,\,b,\,c$ lần lượt là ba nghiệm của phương trình
\[t^3 - t^2 + \frac{2+\sqrt[3]{4}}{12} t - \frac{1}{36} = 0.\]
Giải phương trình này ta được nghiệm \eqref{nghiembac3}. Chứng minh hoàn tất.

Bài toán 9. Cho $a,\,b,\,c$ là ba số thực dương. Chứng minh rằng
\[\frac{a}{b}+\frac{b}{c}+\frac{c}{a}+\frac{(4\sqrt{2}-4)(ab+bc+ca)}{a^2+b^2+c^2} \geqslant 4\sqrt{2}-2+\frac{a^2+b^2+c^2}{ab+bc+ca}.\]

(Nguyễn Văn Huyện)

Lời giải. Chuẩn hóa $p=1$ và áp dụng bổ đề \eqref{(bodehoanvi)} ta đưa bài toán về chứng minh
\[\frac{2(27q^2-9q+1)}{9q^2-2q+(1-3q)\sqrt{q(1-3q)}} + \frac{1}{q} - 6+\frac{(4\sqrt{2}-4)q}{1-2q} \geqslant 4\sqrt{2}-2+\frac{1-2q}{q},\]
hay là
\begin{equation} \label{lab7}
\frac{2(27q^2-9q+1)}{9q^2-2q+(1-3q)\sqrt{q(1-3q)}} +\frac{(4\sqrt{2}-4)q}{1-2q} \geqslant 4\sqrt{2}+2.
\end{equation}
Đặt $x = \sqrt{\frac{1-3q}{q}} \geqslant 0$ thì $q = \frac{1}{x^2+3},$ khi đó bất đẳng thức \eqref{lab7} trở thành
\[\frac{x^3+3x^2+3x+3}{x+1} \geqslant \frac{x^4+4\sqrt{2}x^2+3}{x^2+1}.\]
Xét hiệu hai vế ta được
\[\frac{x^3+3x^2+3x+3}{x+1} - \frac{x^4+4\sqrt{2}x^2+3}{x^2+1} = \frac{2x^2\big(x+1-\sqrt{2}\big)^2}{(x+1)(x^2+1)} \geqslant 0.\]
Đẳng thức xảy ra khi và chỉ khi $a=b=c$ hoặc $a,\,b,\,c$ lần lượt là ba nghiệm của phương trình
\[t^3-t^2+\frac{3+\sqrt{2}}{14}t - \frac{2+3\sqrt{2}}{196} = 0,\]
cụ thể
\[\left\{ \begin{aligned} & a = \frac{1}{3}+\frac{1}{3}\sqrt{\frac{10-6\sqrt{2}}{7}} \cos \left(\frac{1}{3} \arccos \sqrt{\frac{115-27\sqrt{2}}{196}}\right) \\ & b = \frac{1}{3}-\frac{1}{3}\sqrt{\frac{10-6\sqrt{2}}{7}} \sin \left(\frac{\pi}{6}-\frac{1}{3} \arccos \sqrt{\frac{115-27\sqrt{2}}{196}}\right) \\ & c = \frac{1}{3}-\frac{1}{3}\sqrt{\frac{10-6\sqrt{2}}{7}} \sin \left(\frac{\pi}{6}+\frac{1}{3} \arccos \sqrt{\frac{115-27\sqrt{2}}{196}}\right)\end{aligned}\right.\]
Bài toán được chứng minh.

Bài toán 10. Cho ba số thực dương $a,\,b,\,c$ thỏa mãn điều kiện
\[\frac{a}{b} + \frac{b}{c} + \frac{c}{a} = \frac{5}{2}\cdot\frac{a^2+b^2+c^2}{ab+bc+ca}.\]
Chứng minh rằng \[a^2+b^2+c^2 \geqslant 2(ab+bc+ca).\]

 

(Nguyễn Văn Huyện)

Lời giải. Chuẩn hóa $p=1$ từ giả thiết áp dụng bổ đề \eqref{(bodehoanvi)} ta có
\begin{equation} \label{lab10}
\frac{2(27q^2-9q+1)}{9q^2-2q+(1-3q)\sqrt{q(1-3q)}}+\frac{1}{q} - 6 \leqslant \frac{5(1-2q)}{2q}.
\end{equation}
Đặt $x = \sqrt{\frac{1-3q}{q}} \geqslant 0$ thì $q = \frac{1}{x^2+3},$ khi đó \eqref{lab10} trở thành
\[\frac{x^3+3x^2+3x+3}{x+1} \leqslant \frac{5(x^2+1)}{2},\]
tương đương với
\[\frac{(x - 1)(3x^2 + 2x + 1)}{x+1} \geqslant 0.\]
Suy ra $x \geqslant 1,$ hay là
\[(a+b+c)^2 \geqslant 4(ab+bc+ca),\]
hoặc
\[a^2+b^2+c^2 \geqslant 2(ab+bc+ca).\]
Đẳng thức xảy ra khi và chỉ khi $a=b=c$ hoặc
\[\frac{a}{\sqrt{7}-\tan\frac{\pi}{7}} = \frac{b}{\sqrt{7}-\tan\frac{2\pi}{7}} = \frac{c}{\sqrt{7}-\tan\frac{4\pi}{7}}.\]
Chứng minh hoàn tất.

Bài toán 11. Với $k \geqslant 0$ là một số thực cho trước và $a,\,b,\,c$ là ba số thực dương sao cho
\[\frac{a}{b}+\frac{b}{c}+\frac{c}{a} = (k + 1)^2 + \frac{2}{k+1}.\]
Chứng minh rằng
\[a^2 + b^2 + c^2 \leqslant (k^2+1)(ab+bc+ca).\]

 

(Nguyễn Văn Huyện, VMEO IV)

Lời giải. Chuẩn hóa $p=1$ từ giả thiết áp dụng bổ đề \eqref{(bodehoanvi)} ta được
\begin{equation} \label{lab9}
\frac{2(27q^2-9q+1)}{9q^2-2q+(1-3q)\sqrt{q(1-3q)}}+\frac{1}{q} - 6 \leqslant (k + 1)^2 + \frac{2}{k+1}.
\end{equation}
Đặt $x = \sqrt{\frac{1-3q}{q}} \geqslant 0$ thì $q = \frac{1}{x^2+3},$ khi đó \eqref{lab9} trở thành
\[\frac{x^3+3x^2+3x+3}{x+1} \leqslant (k + 1)^2 + \frac{2}{k+1},\]
hay
\[(x+1)^2 + \frac{2}{x+1} \leqslant (k + 1)^2 + \frac{2}{k+1},\]
tương đương với
\[\frac{(x-k)\big[(k+1)x^2+(k^2+4k+3)x+k^2+3k\big]}{(x+1)(k+1)} \leqslant 0.\]
Suy ra $x \leqslant k$ hay là
\[(a+b+c)^2\leqslant (k^2+3)(ab+bc+ca),\]
hoặc
\[a^2 + b^2 + c^2 \leqslant (k^2+1)(ab+bc+ca).\]
Bài toán được chứng minh.

 

Nhận xét. Trường hợp $k = 1$ ta được bài toán rất đẹp sau

Nếu $a,\,b,\,c$ là ba số thực dương thỏa mãn
\[\frac{a}{b}+\frac{b}{c}+\frac{c}{a} = 5,\]
thì
\[a^2 + b^2 + c^2 \leqslant 2(ab+bc+ca).\]

 

(Võ Quốc Bá Cẩn)

3. Các bài toán rèn luyện

Để kết thúc chuyên đề xin được giới thiệu một số bài tập để bạn đọc tự luyện

Bài tập 1. Cho $a,\,b,\,c$ là ba số thực dương. Chứng minh rằng
\[\left(\frac{a}{b}+\frac{b}{c}+\frac{c}{a}\right)^3+\frac{525(ab+bc+ca)}{a^2+b^2+c^2} \geqslant \frac{775}{2}.\]

 

(Nguyễn Văn Huyện)

Bài tập 2. Chứng minh rằng với mọi số thực $k \geqslant 12$ ta luôn có
\[\frac{a}{b}+\frac{b}{c}+\frac{c}{a}+\frac{k(ab+bc+ca)}{(a+b+c)^2} \geqslant 4+2\sqrt{k-12},\]
trong đó $a,\,b,\,c$ là ba số thực dương thay đổi bất kỳ.

 

(Tạ Hồng Quảng)

Bài tập 3. Cho $a,\,b,\,c$ là ba số thực dương. Chứng minh rằng
\[\frac{a}{b}+\frac{b}{c}+\dfrac{c}{a} \geqslant \sqrt{\frac{k(a^2+b^2+c^2)}{(a+b+c)^2}+9-\dfrac{k}{3}},\]
trong đó $k = 54\sqrt[3]{2}.$

 

(Bách Ngọc Thành Công)

 

Bài tập 4. Tìm hằng số $k$ lớn nhất sao cho bất đẳng thức
\[\frac{a}{b}+\frac{b}{c}+\frac{c}{a} \geqslant \sqrt{9+k-\frac{k(ab+bc+ca)}{a^2+b^2+c^2}},\]
luôn đúng với mọi số thực dương $a,\,b,\,c.$

Bài tập 5. Tìm hằng số $k$ lớn nhất sao cho bất đẳng thức
\[\frac{a}{b} + \frac{b}{c} + \frac{c}{a} + k \geqslant \sqrt{\frac{k^2(a^2+b^2+c^2)}{ab+bc+ca}} + 3,\]
luôn đúng với mọi số thực dương $a,\,b,\,c$ bất kỳ.

Bài tập 6. Cho $a,\,b,\,c$ là ba số thực dương. Chứng minh rằng
\[\frac{a}{b} + \frac{b}{c} + \frac{c}{a} \geqslant \frac{a^2+b^2+c^2}{ab+bc+ca} + 2 \sqrt{\frac{a^2+b^2+c^2}{ab+bc+ca}}.\]

Bài tập 7. Cho $a,\,b,\,c$ là ba số thực dương. Chứng minh rằng
\[\frac{a}{b} + \frac{b}{c} + \frac{c}{a} + \frac{k(ab+bc+ca)}{a^2+b^2+c^2+ab+bc+ca} \geqslant 3+ \frac{k}{2},\]
trong đó $k = 2\big(3\sqrt[3]{9}-1\big).$

 

(Nguyễn Văn Huyện)

Bài tập 8. Tìm hằng số $k$ lớn nhất sao cho bất đẳng thức
\[\frac{a}{b} + \frac{b}{c} + \frac{c}{a} + k \geqslant \sqrt{\frac{3k^2(a^2+b^2+c^2)}{(a+b+c)^2}} + 3,\]
luôn đúng với mọi số thực dương $a,\,b,\,c$ bất kỳ.

 

(Phạm Sinh Tân)

Bài tập 9. Tìm hằng số $k$ lớn nhất sao cho bất đẳng thức
\[\frac{a}{b} + \frac{b}{c} + \frac{c}{a} + k\left ( \frac{ab+bc+ca}{a^2+b^2+c^2} \right )^2 \geqslant  3+k,\]
luôn đúng với mọi số thực dương $a,\,b,\,c$ bất kỳ.

Bài tập 10. Với $k \geqslant 1$ là một số thực cho trước và $a,\,b,\,c$ là ba số thực dương sao cho
\[\frac{a}{b} + \frac{b}{c} + \frac{c}{a} = \frac{(k^2+9)(a^2+b^2+c^2)}{(a+b+c)^2}.\]
Chứng minh rằng
\[a^2+b^2+c^2 \leqslant (k^2+1)(ab+bc+ca).\]

 

(Nguyễn Văn Huyện)

4. Tài liệu tham khảo

 

  1. Võ Quốc Bá Cẩn, Chuyên Đề Bất Đẳng Thức Hiện Đại, 2008.
  2. Diễn đàn toán học: http://diendantoanhoc.net
  3. Art of Problem Solving: http://artofproblemsolving.com

File gửi kèm


Nguyen Van Huyen
Ho Chi Minh City University Of Transport

#2
Gachdptrai12

Gachdptrai12

    Thượng sĩ

  • Điều hành viên THCS
  • 280 Bài viết

 bài đọc này bổ ích ghê hình như bài của VMEO4 mở rộng của bổ đề đúng ko ạ ?? thank anh jup em có k thức mới :v

em có bài này nè cũng hay

cho a,b,c >0 chứng minh $a^{3}+b^{3}+c^{3}+3abc\frac{a^{2}b+b^{2}c+c^{2}a}{ab^{2}+bc^{2}+ca^{2}}\geq ab(a+b)+bc(b+c)+ca(c+a)$



#3
hoanghuutai39

hoanghuutai39

    Binh nhì

  • Thành viên mới
  • 13 Bài viết

Bổ đề cồng kềnh quá. Nhớ mà dùng cũng là một vấn đề lớn :D
 


Website tải tài liệu môn Toán THPT cực hay 
 

http://www.toanmath.com/


#4
Nguyenhuyen_AG

Nguyenhuyen_AG

    Trung úy

  • Thành viên nổi bật 2016
  • 945 Bài viết

 bài đọc này bổ ích ghê hình như bài của VMEO4 mở rộng của bổ đề đúng ko ạ ?? thank anh jup em có k thức mới :v

em có bài này nè cũng hay

cho a,b,c >0 chứng minh $a^{3}+b^{3}+c^{3}+3abc\frac{a^{2}b+b^{2}c+c^{2}a}{ab^{2}+bc^{2}+ca^{2}}\geq ab(a+b)+bc(b+c)+ca(c+a)$

 

Bài này anh thấy nó đâu có liên quan gì với bổ đề đâu, em có thể đăng nó trong topic tiếp sức bất đẳng thức sẽ có người giải. :)

 

Bổ đề cồng kềnh quá. Nhớ mà dùng cũng là một vấn đề lớn :D

 

Các bài toán trong này không dùng cho thi Olympic em à. Thật sự thì chúng quá khó đặc biệt là các bài 5, 6, 7, 8, 9 các hằng số $k$ ở đó đều là các hằng số tốt nhất. Bài này anh nghĩ các bạn thích nghiên cứu về bất đẳng thức đọc sẽ thích, còn các bạn muốn thi HSG thì đọc chơi cho biết thôi, đừng bận tâm nhiều quá vì đi thi chả ai cho các dạng bài này. :)


Nguyen Van Huyen
Ho Chi Minh City University Of Transport

#5
Nesbit

Nesbit

    ...let it be...

  • Quản lý Toán Ứng dụng
  • 2409 Bài viết

Cảm ơn Huyện về món quà ý nghĩa  :namtay  Bài viết rất hay, và trình bày cũng rất đẹp mắt nữa. Anh em còn chờ gì nữa mà không share lên FB :D

 

Chúc mọi người năm mới sức khỏe và gặp nhiều may mắn !


Không đọc tin nhắn nhờ giải toán.

 

Góp ý về cách điều hành của mod

 

 


#6
Nesbit

Nesbit

    ...let it be...

  • Quản lý Toán Ứng dụng
  • 2409 Bài viết

Anh có một góp ý nhỏ.

 

 

Nhận xét.

 

  1. ...
  2. Đây là một kết quả rất chặt với vô số các trường hợp để đẳng thức xảy ra và cũng là dạng chặt nhất trong lớp các bài toán có dạng \[f\left(\frac{a}{b},\,\frac{b}{c},\,\frac{c}{a},\,a+b+c,\,ab+bc+ca\right) \geqslant 0.\]

Phần in đậm không biết em đọc được ở đâu hay là em chứng minh được, nhưng nó không đúng đâu. Em nên bỏ đi (hoặc đưa ra chứng minh) để bài viết được chặt chẽ.


Không đọc tin nhắn nhờ giải toán.

 

Góp ý về cách điều hành của mod

 

 


#7
nhatkinan

nhatkinan

    Hạ sĩ

  • Thành viên
  • 56 Bài viết

cảm ơn bạn 

bạn có nhiều bài viết rất hay



#8
toanhoc2017

toanhoc2017

    Thiếu úy

  • Banned
  • 628 Bài viết
mong co nhieu bai tap dang nay nhe




0 người đang xem chủ đề

0 thành viên, 0 khách, 0 thành viên ẩn danh